Prove that a fuction is continous and differentiable everywhere, but not at f'=0

  • Thread starter Thread starter charmedbeauty
  • Start date Start date
  • Tags Tags
    Differentiable
Click For Summary
The function f(x) = x²sin(1/x) for x≠0 and f(0) = 0 is continuous and differentiable everywhere, yet its derivative f' is not continuous at x=0. This serves as a counterexample demonstrating that a function can be differentiable everywhere without having a continuous derivative. The definitions of continuity and differentiability confirm that f is continuous at all points, including x=0. However, the behavior of f' near x=0 reveals its discontinuity due to the oscillatory nature of sin(1/x). Thus, while f is smooth, its derivative exhibits a break at the origin.
charmedbeauty
Messages
266
Reaction score
0

Homework Statement



Prove that the function f:ℝ→ℝ, given by

f(x)={x2sin(1/x) if x≠0, 0 if x=0}

is continuous and differentiable everywhere, but that f' is not continuous at 0.



Homework Equations





The Attempt at a Solution



I thought if a function was differentiable everywhere then it was continuous everywhere?

If not what should I do??
 
Last edited:
Physics news on Phys.org
Hi charmedbeauty! :smile:

I guess this is exactly the counter example that shows that differentiability (even everywhere) does not imply continuity of the derivative.

What you would need to do is to check the definitions of continuity and differentiability.
Do you have those handy?

I'll help you along.
A function f is continuous at point c iff ##\lim\limits_{x \to c} f(x)=f(c)##.
Does that hold for c=0 in your case?
 
Last edited:
Differentiability DOES imply continuity. f IS continuous. It's f' that is discontinuous.
 
f' exists for all values of x which implies that f is differentiable and hence f is continuous too
f' not being continuous is a different matter
 
cos(1/x) is not continuous at x=0
 
Question: A clock's minute hand has length 4 and its hour hand has length 3. What is the distance between the tips at the moment when it is increasing most rapidly?(Putnam Exam Question) Answer: Making assumption that both the hands moves at constant angular velocities, the answer is ## \sqrt{7} .## But don't you think this assumption is somewhat doubtful and wrong?

Similar threads

Replies
6
Views
2K
  • · Replies 5 ·
Replies
5
Views
2K
Replies
26
Views
2K
  • · Replies 8 ·
Replies
8
Views
1K
  • · Replies 39 ·
2
Replies
39
Views
6K
  • · Replies 9 ·
Replies
9
Views
6K
  • · Replies 2 ·
Replies
2
Views
1K
  • · Replies 7 ·
Replies
7
Views
2K
  • · Replies 17 ·
Replies
17
Views
2K
Replies
1
Views
2K